Not signed in (Sign In)

Vanilla 1.1.9 is a product of Lussumo. More Information: Documentation, Community Support.

  1.  

    http://mathoverflow.net/questions/49728/stationary-sets

    It seems like the question was mostly closed because of formatting. After consulting with my local set theorist, I reopened this question (he said it was not hard, but non-trivial enough to be okay). I'm going to remove the comments referring to formatting, but I'm pasting the whole comment thread below for posterity.

    This was closed quickly while the question was in an illegible form. I have fixed the TeX. – Robin Chapman yesterday

    I'm sorry but why this topic was closed? – sonicyouth yesterday

    You made a TeX-mistake and the question was not displayed apart from the first 4 characters. Robin Chapman fixed the TeX so that everybody could read the question. Before that I thought it is spam and voted to close it. I am now voting to reopen it. There is only one vote missing before it is reopened. – Andreas Thom yesterday

    The question was not readable. I think it read something like Let {S_i and that was it. I am sorry, but I should have checked the LaTeX like Robin and attempted to fix it myself. I voted to reopen and encourage others to do so. – Tony Huynh yesterday

    Thank you, and sorry for my mistakes:) – sonicyouth yesterday

    Before people vote to reopen: This is an elementary question, not appropriate for the site. – Andres Caicedo yesterday

    I completely agree with Andres. This question belongs to math.stackexchange.com and not here. – Asaf Karagila yesterday

    I just want to know the answer to this question, I am not sure about the meaning of "elementary" posed by Andres Caicedo, is it a question too easy to answer? – sonicyouth yesterday

    @sonicyouth: Elementary in the sense that MO is dealing with research level questions, and this is most certainly not such question. This is an advanced undergrad (or grad) level course material. As I suggested in my other comment, math.stackexchange.com is a far more suitable location for this question. – Asaf Karagila yesterday

  2.  
    Anton, the question is trivial. Was asked on the sister site, and I answered it in line and a half. The OP admitted "It is really elementary :-)"
  3.  
    http://math.stackexchange.com/questions/14747/is-the-intersection-of-a-decreasing-sequence-of-countably-many-stationary-subsets (It is longer than line and a half, yes, because it is the sister site, so I give more details than would be needed here.)
  4.  

    From the comment thread:

    Anton, I do not understand the distrust to our judgement.

    At the time I reopened the question, there were three votes to reopen and there was evidence that the question was closed because of formatting. The question is clearly posed and well-defined. A question having an elementary solution is no reason for it to be closed, and consulting a set theorist suggested that the question is not completely trivial. One comment said

    This is an advanced undergrad (or grad) level course material.

    If this is accurate, I don't think it makes sense to close it based solely on "level." Many of the best questions and answers on MO are, strictly speaking, elementary. Maybe I'm misunderstanding how elementary the question is. @Andres: In your answer, you cite a blog post you wrote about this kind of thing (or at least related material). This suggests to me that (1) you found it worth writing a blog post about, and (2) since it looks like a teaching blog post, you're likely to underestimate the complexity of related problems. The basic question is whether this is a question a professional mathematician in another field could reasonably have when learning set theory.

    I don't know any set theory, but given the meta-data I have right now, I don't regret reopening the question. Of course, I do not object to people more knowledgeable than me doing a better job. I just haven't seen sufficient evidence that the question really should be closed.